Solution to Problem 681 | Midspan Deflection

Problem 681
Show that the midspan value of EIδ is (wob/48)(L3 - 2Lb2 + b3) for the beam in part (a) of Fig. P-681. Then use this result to find the midspan EIδ of the loading in part (b) by assuming the loading to exceed over two separate intervals that start from midspan and adding the results.
 

681a-simple-beam-uniform-load-general.jpg       681b-simple-beam-uniform-load-specific.jpg

 

Solution 681